Inscription / Connexion Nouveau Sujet
Niveau terminale
Partager :

Exercice sur les congruences

Posté par
alggie
15-02-19 à 12:25

Bonjour, voici un exercice  que je n'arrive pas à terminer... Merci de votre aide

Exercice 1 :

Soit x un entier naturel non nul, multiple de 6. Soit p un nombre premier divisant x^2+x+1. On sait que p s'écrit sous la forme 6n+1 ou 6n+5 et que x^3 est congru à 1 modulo p et que x^6n est congru à 1 modulo p.

1. On suppose que p=6n+5, montrer que x est congru à 1 modulo p. Que vaut alors p? On pourra utiliser le petit théorème de Fermat.
2. Déduire de ce qui précède que p divise x^2+x+1 et est de la forme 6n+1.
3. Soit un entier N>3. On pose x=N! (! la fonction factorielle). Prouver que p>N. Montrer qu'il existe une infinité de nombres premiers de la forme 6n+1.

Ce que j'ai fait :

J'ai montré que x est congru à 1 modulo p. Mais je n'arrive pas à faire le reste de la question et les autres questions.

Merci beaucoup pour votre aide!

Posté par
carpediem
re : Exercice sur les congruences 15-02-19 à 12:39

salut

énoncé pas clair du tout ...

alggie @ 15-02-2019 à 12:25

Soit x un entier naturel non nul, multiple de 6. Soit p un nombre premier divisant x^2+x+1. On sait que p s'écrit sous la forme 6n+1 ou 6n+5 et que x^3 est congru à 1 modulo p et que x^6n est congru à 1 modulo p.

1. On suppose que p=6n+5, montrer que x est congru à 1 modulo p. Que vaut alors p? On pourra utiliser le petit théorème de Fermat.
2. Déduire de ce qui précède que p divise x^2+x+1 et est de la forme 6n+1.

Posté par
alggie
re : Exercice sur les congruences 15-02-19 à 14:47

Bonjour,

Oui je sais que ce n'est pas logique mais c'est ce qu'il y a dans mon DM...

Merci

Posté par
Yzz
re : Exercice sur les congruences 15-02-19 à 17:40

Salut,

Vraiment pas clair !!!

Citation :
1. On suppose que p=6n+5, montrer que x est congru à 1 modulo p. Que vaut alors p? On pourra utiliser le petit théorème de Fermat.
2. Déduire de ce qui précède que p divise x^2+x+1 et est de la forme 6n+1.


Citation :
c'est ce qu'il y a dans mon DM...
Est-ce au c'est écrit exactement ça, mot à mot, du début à la fin de l'exo ?

Posté par
Sylvieg Moderateur
re : Exercice sur les congruences 15-02-19 à 17:58

Bonjour,

Citation :
On sait que p s'écrit sous la forme 6n+1 ou 6n+5 et que x^3 est congru à 1 modulo p et que x^6n est congru à 1 modulo p.
On le sait car écrit exactement ainsi dans l'énoncé ou car démontré auparavant ???

Posté par
alggie
re : Exercice sur les congruences 15-02-19 à 18:06

Yzz @ 15-02-2019 à 17:40

Est-ce au c'est écrit exactement ça, mot à mot, du début à la fin de l'exo ?


Pour cette partie là, oui. Je ne comprends pas non plus pourquoi on me demande de déduire quelque chose d'admis dès le début de l'exercice...

Posté par
alggie
re : Exercice sur les congruences 15-02-19 à 18:09

Sylvieg @ 15-02-2019 à 17:58

Bonjour,
Citation :
On sait que p s'écrit sous la forme 6n+1 ou 6n+5 et que x^3 est congru à 1 modulo p et que x^6n est congru à 1 modulo p.
On le sait car écrit exactement ainsi dans l'énoncé ou car démontré auparavant ???
Bonsoir, je l'ai démontré dans une première partie. J'avais les questions :

1. En travaillant modulo 6, montrer que p est différent de 2 et 3 et s'écrit sous la forme 6n+1 ou 6n+5 avec n>=1.

2. Montrer que x^3=1(p) (le = est pour la congruence), puis que x^{6n}=1(p) (le = est pour la congruence) pour tout entier naturel n>=1. On sait que : x^3-1=(x-1)(x²+x+1).

Posté par
alggie
re : Exercice sur les congruences 18-02-19 à 09:18

Bonjour,

Pour la question 2, j'ai dis que si x=1(p) (= est congru) alors x-1=pk avec k un entier relatif et donc p=\frac {x-1}{k}.
Or : x=6n avec n un entier relatif. D'où : p=6n+1.
Le problème de cette réponse c'est que le k disparaît comme par magie...

Merci à vous.

Posté par
alggie
re : Exercice sur les congruences 18-02-19 à 09:21

alggie @ 18-02-2019 à 09:18

Bonjour,

Pour la question 2, j'ai dis que si x=1(p) (= est congru) alors x-1=pk avec k un entier relatif et donc p=\frac {x-1}{k}.
Or : x=6n avec n un entier relatif. D'où : p=6n+1.
Le problème de cette réponse c'est que le k disparaît comme par magie...

Merci à vous.


Erreur de ma part : je suis passée du x-1 au x+1. Cette réponse est donc fausse.

Posté par
carpediem
re : Exercice sur les congruences 18-02-19 à 09:52

en fait j'aimerai bien voir la question 1/ ...

alggie @ 15-02-2019 à 12:25

Ce que j'ai fait :

J'ai montré que x est congru à 1 modulo p.

Posté par
alggie
re : Exercice sur les congruences 18-02-19 à 17:43

carpediem @ 18-02-2019 à 09:52

en fait j'aimerai bien voir la question 1/ ...
Si tu parles de la réponse : On sait que : x^{6n}≡1[p] et que x^3≡1[p] donc x^{6n}×x^3×x^1≡1[p]⇔x^1≡1[p].

On en déduit que : x^1≡1[p] soit  x≡1[p].

Posté par
alggie
re : Exercice sur les congruences 18-02-19 à 17:43

alggie @ 18-02-2019 à 17:43

carpediem @ 18-02-2019 à 09:52

en fait j'aimerai bien voir la question 1/ ...
Si tu parles de la réponse : On sait que : x^{6n}≡1[p] et que x^3≡1[p] donc x^{6n}×x^3×x^1≡1[p]⇔x^1≡1[p].

On en déduit que : x^1≡1[p] soit  x≡1[p].


Les parties bizarres de l'équation, ce sont les = congrus.

Posté par
carpediem
re : Exercice sur les congruences 18-02-19 à 18:10



et si tu n'arrives pas à faire un congru à il suffit de mettre un = ....

en latex : \equiv a \equiv b

Posté par
alggie
re : Exercice sur les congruences 18-02-19 à 18:26

carpediem @ 18-02-2019 à 18:10



et si tu n'arrives pas à faire un congru à il suffit de mettre un = ....

en latex : \equiv a \equiv b
Merci je note

Posté par
carpediem
re : Exercice sur les congruences 18-02-19 à 18:37

j'eu aimé que tu notas et recopias ton msg afin de le rendre lisible ...

Posté par
alggie
re : Exercice sur les congruences 19-02-19 à 09:50

carpediem @ 18-02-2019 à 18:37

j'eu aimé que tu notas et recopias ton msg afin de le rendre lisible ...


Revoilà le message (bien lisible cette fois ☻) :

Soit p un nombre premier ne divisant pas x, un entier naturel non nul. D'après le petit théorème de Fermat, on a : x^{p-1}\equiv 1 [p].

Or : p=6n+5.
Soit : x^{6n+5-1} \equiv 1 [p]
x^{6n+4}\equiv 1 [p]

Or : x^{6n+4}=x^{6n}\times x^3 \times x^1
Soit : x^{6n}\times x^3 \times x^1 \equiv 1 [p]

On sait que : x^{6n} \equiv 1 [p] et x^3 \equiv 1 [p] donc x^{6n}\times x^3 \times x^1\equiv 1 [p] \Leftrightarrow x^1 \equiv 1[p].

On en déduit que : x^1 \equiv 1 [p] d'où : x \equiv 1 [p].

Posté par
carpediem
re : Exercice sur les congruences 19-02-19 à 10:11

ok merci  ...

mais franchement je ne comprends toujours pas la logique ...

en particulier si on sait que x^3 = 1 [p] alors il est évident qu'il en est de même de (x^3)^k donc en particulier de x^(6n)

Posté par
alggie
re : Exercice sur les congruences 20-02-19 à 09:45

carpediem @ 19-02-2019 à 10:11

ok merci  ...

mais franchement je ne comprends toujours pas la logique ...

en particulier si on sait que x^3 = 1 [p] alors il est évident qu'il en est de même de (x^3)^k donc en particulier de x^(6n)
Donc ma démonstration est fausse ? Ou c'est la logique de l'exercice qui est étrange ?

Posté par
carpediem
re : Exercice sur les congruences 20-02-19 à 12:51

la logique de l'exercice est de montrer que si on prend p = 6n + 5 et vérifiant les conditions de l'énoncé ... ben alors ça ne marche pas et que p = 6n + 1

puis on en démontre l'infinité ...

Posté par
alggie
re : Exercice sur les congruences 21-02-19 à 09:20

carpediem @ 20-02-2019 à 12:51

la logique de l'exercice est de montrer que si on prend p = 6n + 5 et vérifiant les conditions de l'énoncé ... ben alors ça ne marche pas et que p = 6n + 1

puis on en démontre l'infinité ...


Donc je dois :

1. Montrer que p=6n+5 ne divise pas x²+x+1.
2. Montrer que p=6n+1 divise x²+x+1.
3. En déduire que p=6n+1.
4. Montrer qu'il existe une infinité de nombres premiers de la forme 6n+1.

C'est ça ??

Merci

Posté par
alggie
re : Exercice sur les congruences 21-02-19 à 09:27

alggie @ 21-02-2019 à 09:20

carpediem @ 20-02-2019 à 12:51

la logique de l'exercice est de montrer que si on prend p = 6n + 5 et vérifiant les conditions de l'énoncé ... ben alors ça ne marche pas et que p = 6n + 1

puis on en démontre l'infinité ...


Donc je dois :

1. Montrer que p=6n+5 ne divise pas x²+x+1.
2. Montrer que p=6n+1 divise x²+x+1.
3. En déduire que p=6n+1.
4. Montrer qu'il existe une infinité de nombres premiers de la forme 6n+1.

C'est ça ??

Merci


Sans oublier le p>N...

Posté par
carpediem
re : Exercice sur les congruences 21-02-19 à 09:48

vu mon premier msg je laisserai tomber cet exo et j'irai voir mon prof pour lui demander ce qui ne va pas ...

Posté par
alggie
re : Exercice sur les congruences 24-02-19 à 11:31

carpediem @ 21-02-2019 à 09:48

vu mon premier msg je laisserai tomber cet exo et j'irai voir mon prof pour lui demander ce qui ne va pas ...


Le problème c'est que ce sont les vacances et qu'on est censé le faire pour la rentrée...

J'ai beau essayé de prouver que p=6n+1 ça ne donne jamais rien de concret. Et je ne parle même pas de l'histoire du n!.

Merci quand même

Posté par
lake
re : Exercice sur les congruences 24-02-19 à 13:23

Bonjour,

On suppose donc que p premier divise x^2+x+1

  x^2+x+1\equiv 0\;\;[p]\Longrightarrow x^3+x^2+x\equiv 0\;\;[p]\Longrightarrow x^3\equiv -x^2-x\equiv 1\;\;[p]

  et x^{6n}\equiv (x^3)^{2n}\equiv 1\;\;[p]

  et ceci sans hypothèses supplémentaires sur x.

1) On suppose p=6n+5

  x^p\equiv x^{6n+5}\equiv x^{6n}.x^3.x^2\equiv x^2\;\;[p]

   et d'après Fermat x^p\equiv x\;\;[p]

D'où x^2\equiv x\;\;[p]

On en déduit que x^3\equiv x^2\equiv x\equiv 1\;\;[p]

Donc x^2+x+1\equiv 3\;\;[p] et 3\equiv 0\;\;[p]

   donc p=3

2) Or si x est multiple de 6,  x^2+x+1\equiv 1\;\;[3]

   Avec les hypothèses de l'énoncé, à savoir:

    - x non nul multiple de 6.

    - p premier divise x^2+x+1,

  on a montré que p est de la forme 6n+1

3) Avec N>3, on a bien N! multiple non nul de 6.

   Supposons p\leq N alors p divise (N!)^2+N! et (N!)^2+N!+1 donc p divise leur différence 1.

  On en déduit que p>N

  On a montré que tout nombre premier qui divise (N!)^2+(N!)+1 (et il en existe: la décomposition en facteurs premiers d'un nombre) est de la forme 6n+1 et est plus grand que N.

  Ce qui signifie que pour un nombre premier p=6n+1 donné, on peut toujours en trouver un qui lui est supérieur: un diviseur premier de (p!)^2+p!+1 qui sera aussi de la forme 6k+1.


Posté par
alggie
re : Exercice sur les congruences 01-03-19 à 12:07

lake @ 24-02-2019 à 13:23

Bonjour,

On suppose donc que p premier divise x^2+x+1

  x^2+x+1\equiv 0\;\;[p]\Longrightarrow x^3+x^2+x\equiv 0\;\;[p]\Longrightarrow x^3\equiv -x^2-x\equiv 1\;\;[p]

  et x^{6n}\equiv (x^3)^{2n}\equiv 1\;\;[p]

  et ceci sans hypothèses supplémentaires sur x.

1) On suppose p=6n+5

  x^p\equiv x^{6n+5}\equiv x^{6n}.x^3.x^2\equiv x^2\;\;[p]

   et d'après Fermat x^p\equiv x\;\;[p]

D'où x^2\equiv x\;\;[p]

On en déduit que x^3\equiv x^2\equiv x\equiv 1\;\;[p]

Donc x^2+x+1\equiv 3\;\;[p] et 3\equiv 0\;\;[p]

   donc p=3

2) Or si x est multiple de 6,  x^2+x+1\equiv 1\;\;[3]

   Avec les hypothèses de l'énoncé, à savoir:

    - x non nul multiple de 6.

    - p premier divise x^2+x+1,

  on a montré que p est de la forme 6n+1

3) Avec N>3, on a bien N! multiple non nul de 6.

   Supposons p\leq N alors p divise (N!)^2+N! et (N!)^2+N!+1 donc p divise leur différence 1.

  On en déduit que p>N

  On a montré que tout nombre premier qui divise (N!)^2+(N!)+1 (et il en existe: la décomposition en facteurs premiers d'un nombre) est de la forme 6n+1 et est plus grand que N.

  Ce qui signifie que pour un nombre premier p=6n+1 donné, on peut toujours en trouver un qui lui est supérieur: un diviseur premier de (p!)^2+p!+1 qui sera aussi de la forme 6k+1.




Merci beaucoup pour ce message

J'ai tout de même 2 questions :

2) Pourquoi en déduit-on que p=6n+1 ? Est-ce parce qu'on sait que p=6n+1 ou p=6n+5. Or, si p=6n+5 la divisibilité avec x²+x+1 est remise en cause donc p=6n+1 ?

3) Là non plus je ne comprends pas comment on déduit que p>N.

Encore merci

Posté par
lake
re : Exercice sur les congruences 01-03-19 à 13:38

1) 2) p est de la forme 6n+1 ou 6n+5 (p=2 et p=3 sont à éliminer car si x est multiple de 6,alors x^2+x+1 n'est ni multiple de 2, ni multiple de 3)

  En supposant que p=6n+5, on a montré que nécessairement, p=3 ce qui est doublement absurde puisque pas de la forme 6n+5 et voir aussi la parenthèse au dessus.

  Donc p est de la forme 6n+1

3)N!  avec N>3 est bien un multiple de 6 et on est dans les hypothèses de 1) et 2).

  En supposant p\leq N, p  divise N!=1\times 2\times 3\cdots \times (N-1)\times N  (p figure quelque part dans le produit de facteurs)

    Donc p divise (N!)^2+N!

et en se replaçant dans les hypothèses de 1) et 2) avec x=N!, c'est à dire p divise x^2+x+1=(N!)^2+N!+1, alors p divise la différence (N!)^2+N!+1-[(N!)^2+N!]=1

  ce qui est absurde; donc p>N



Vous devez être membre accéder à ce service...

Pas encore inscrit ?

1 compte par personne, multi-compte interdit !

Ou identifiez-vous :


Rester sur la page

Inscription gratuite

Fiches en rapport

parmi 1675 fiches de maths

Désolé, votre version d'Internet Explorer est plus que périmée ! Merci de le mettre à jour ou de télécharger Firefox ou Google Chrome pour utiliser le site. Votre ordinateur vous remerciera !